Last visit was: 18 Nov 2025, 16:09 It is currently 18 Nov 2025, 16:09
Close
GMAT Club Daily Prep
Thank you for using the timer - this advanced tool can estimate your performance and suggest more practice questions. We have subscribed you to Daily Prep Questions via email.

Customized
for You

we will pick new questions that match your level based on your Timer History

Track
Your Progress

every week, we’ll send you an estimated GMAT score based on your performance

Practice
Pays

we will pick new questions that match your level based on your Timer History
Not interested in getting valuable practice questions and articles delivered to your email? No problem, unsubscribe here.
Close
Request Expert Reply
Confirm Cancel
605-655 Level|   Weaken|               
avatar
ttwang56
Joined: 26 Jan 2008
Last visit: 18 Sep 2008
Posts: 14
Own Kudos:
201
 [186]
Posts: 14
Kudos: 201
 [186]
29
Kudos
Add Kudos
156
Bookmarks
Bookmark this Post
Most Helpful Reply
User avatar
GyanOne
Joined: 24 Jul 2011
Last visit: 16 Nov 2025
Posts: 3,222
Own Kudos:
1,690
 [34]
Given Kudos: 33
Status: World Rank #4 MBA Admissions Consultant
Expert
Expert reply
Posts: 3,222
Kudos: 1,690
 [34]
26
Kudos
Add Kudos
8
Bookmarks
Bookmark this Post
User avatar
pqhai
User avatar
Retired Moderator
Joined: 16 Jun 2012
Last visit: 26 Nov 2015
Posts: 867
Own Kudos:
8,883
 [26]
Given Kudos: 123
Location: United States
Posts: 867
Kudos: 8,883
 [26]
21
Kudos
Add Kudos
5
Bookmarks
Bookmark this Post
General Discussion
User avatar
nmohindru
Joined: 06 Apr 2008
Last visit: 06 Jul 2013
Posts: 157
Own Kudos:
488
 [1]
Given Kudos: 1
Posts: 157
Kudos: 488
 [1]
1
Kudos
Add Kudos
Bookmarks
Bookmark this Post
ttwang56
The proposal to hire ten new police officers in Middletown is quite foolish. There is sufficient funding to pay the salaries of the new officers, but not the salaries of additional court and prison employees to process the increased caseload of arrests and convictions that new officers usually generate. Which of the following, if true, will most seriously weaken the conclusion drawn above?
a. Studies had shown that an increase in city’s place force does not necessarily reduce crime
b. When one major city increased its police force by 19% last year, there were 40% more arrests and 13% more convictions
c. If funding for the new police officers’ salaries is approved, support for other city services will have to be reduced during the next fiscal year
d. In most US cities, not all arrests result in convictions, and not all convictions result in prison terms
e. Middletown’s ratio of police officers to citizens has reached a level at which an increase in the number of officers will have a deterrent effect on crime

IMO E)

If crime will be less then number of convictions/arrests will be less as well
avatar
ttwang56
Joined: 26 Jan 2008
Last visit: 18 Sep 2008
Posts: 14
Own Kudos:
Posts: 14
Kudos: 201
Kudos
Add Kudos
Bookmarks
Bookmark this Post
doesn't that support the argument though?
User avatar
rao
Joined: 20 Sep 2006
Last visit: 06 Oct 2011
Posts: 473
Own Kudos:
320
 [6]
Given Kudos: 7
Posts: 473
Kudos: 320
 [6]
5
Kudos
Add Kudos
1
Bookmarks
Bookmark this Post
ttwang56
doesn't that support the argument though?


Nope ... Its is weakening the argument here the flow:

More police --> Less Crime --> less arrests --> Less employes in court and prison to process the caseload of arrests. hence the proposal is not bad et all.
User avatar
sguptashared
Joined: 11 Jun 2011
Last visit: 27 Dec 2013
Posts: 98
Own Kudos:
252
 [2]
Given Kudos: 33
Status:Dream big, work hard, and drink gallons of beer!
Concentration: Finance, General Management
GMAT Date: 10-01-2011
WE:Web Development (Consulting)
Posts: 98
Kudos: 252
 [2]
2
Kudos
Add Kudos
Bookmarks
Bookmark this Post
E is Right. If crime is reduced, salaries for addition court and prison employees are not needed, so the decision to hire new officers is not really foolish.
User avatar
VKat
Joined: 15 Jun 2016
Last visit: 16 Oct 2025
Posts: 91
Own Kudos:
Given Kudos: 741
Posts: 91
Kudos: 24
Kudos
Add Kudos
Bookmarks
Bookmark this Post
I ma confused in option A and E, could you please explain?
User avatar
sayantanc2k
Joined: 14 Dec 2013
Last visit: 09 Dec 2022
Posts: 2,393
Own Kudos:
15,523
 [1]
Given Kudos: 26
Location: Germany
Schools:
GMAT 1: 780 Q50 V47
WE:Corporate Finance (Pharmaceuticals and Biotech)
Expert
Expert reply
Schools:
GMAT 1: 780 Q50 V47
Posts: 2,393
Kudos: 15,523
 [1]
Kudos
Add Kudos
1
Bookmarks
Bookmark this Post
VKat
I ma confused in option A and E, could you please explain?

A and E states the opposite things.
A: increase in police force ---> no decrease in crime. Thus the argument is strengthened.
E: increase in police force ---> decrease in crime. Thus the argument is weakened.
User avatar
ravigupta2912
User avatar
Current Student
Joined: 26 May 2019
Last visit: 16 Feb 2025
Posts: 726
Own Kudos:
291
 [1]
Given Kudos: 84
Location: India
GMAT 1: 650 Q46 V34
GMAT 2: 720 Q49 V40
GPA: 2.58
WE:Consulting (Consulting)
Products:
Kudos
Add Kudos
1
Bookmarks
Bookmark this Post
A. Studies had shown that an increase in city’s place force does not necessarily reduce crime -- Strengthener at worst. Irrelevant if assumed word by word since this isn't saying anything to weaken the conclusion. Okay, crimes aren't reduced but are increasing arrests and convictions done? We don't know. Eliminate.

B. When one major city increased its police force by 19% last year, there were 40% more arrests and 13% more convictions -- Strengthener. Eliminate.

C. If funding for the new police officers’ salaries is approved, support for other city services will have to be reduced during the next fiscal year -- While this attacks the conclusion but it doesn't attack the reasoning behind the argument (increased load of arrests and convictions), therefore, eliminate.

D. In most US cities, not all arrests result in convictions, and not all convictions result in prison terms -- Trap. "not all" could be 99 to 1 or 90 to 1. Both could mean an increased caseload. Eliminate.

E. Middletown’s ratio of police officers to citizens has reached a level at which an increase in the number of officers will have a deterrent effect on crime -- Correct. This gives us an alternate reason to believe that increased arrests and convictions may not happen since the crime will reduce due to increased presence of police officers.
User avatar
CrackverbalGMAT
User avatar
Major Poster
Joined: 03 Oct 2013
Last visit: 16 Nov 2025
Posts: 4,844
Own Kudos:
8,945
 [2]
Given Kudos: 225
Affiliations: CrackVerbal
Location: India
Expert
Expert reply
Posts: 4,844
Kudos: 8,945
 [2]
1
Kudos
Add Kudos
1
Bookmarks
Bookmark this Post
The proposal to hire ten new police officers in Middletown is quite foolish. There is sufficient funding to pay the salaries of the new officers, but not the salaries of additional court and prison employees to process the increased caseload of arrests and convictions that new officers usually generate.

Which of the following, if true, will most seriously weaken the conclusion drawn above?

At a 700 level, you can surely expect that the question either will have a trick option that will look really good/bad or will have something complex in the options that will eat into the time you will take. Let’s see if this question has either...

Remember! The most important task is to correctly identify the conclusion. In a “Weaken the argument” or “Strengthen the argument” question, the word “argument” is a synonym for conclusion. So, if you are not sure about what the conclusion is, how do you strengthen or weaken it?

I hope you’ve correctly identified that the author has concluded that it is a bad idea to hire ten new police officers.

Second, it is always a good idea to clearly (and simply) state what the task at hand is.

What do we need to do in this question? Weaken!

Simply put, we need to find an option that will help us show that it is a good idea to hire ten new police officers. Every option you look at, you need to ask yourself, “Will this option will help us show that it is a good idea to hire ten new police officers?”

Let’s look at the options…


A. Studies had shown that an increase in city’s police force does not necessarily reduce crime

If an increase in the number of police officers will not necessarily reduce crime, then is it a good idea to hire ten new police officers? Not at all! Why even bother?! Eliminate!

B. When one major city increased its police force by 19% last year, there were 40% more arrests and 13% more convictions

The argument tells us that having more officers on the force increases the caseload of arrests and this increases the amount of money shelled out as salary for the additional court and prison employees. Option B tell us that increasing the work force increased the arrests and convictions. This means that there will be an increase in the money spent. In other words, this actually strengthens the argument! Hard pass!

C. If funding for the new police officers’ salaries is approved, support for other city services will have to be reduced during the next fiscal year

You could say that what happens to the support to the other city services is of no concern to us. But let’s look a little closer. I suppose, we could safely say that additional court employees and prison employees are a part of “other city services”. Even if this is not so, if hiring more police officers will cause trouble in other areas of the city, is it a good idea to hire more police officers? No. Eliminate!

D. In most US cities, not all arrests result in convictions, and not all convictions result in prison terms

These kinds of options tend to confuse the most! True… not all arrests result in convictions, and not all convictions result in prison terms. Taking this into consideration, you could say that since not all arrests result in convictions, and not all convictions result in prison terms, the additional salaries will not necessarily have to be paid. Therefore, it is a good idea to hire more police officers.

But wait…

“Not all” result in convictions and prison terms. This means that some or maybe even many do result in convictions and prison terms, right?

This means that the amount of money the city will have to spend will increase and this means that hiring more police officers is not a good idea. So here's the trick option we spoke of at the beginning. Let’s eliminate!


E. Middletown’s ratio of police officers to citizens has reached a level at which an increase in the number of officers will have a deterrent effect on crime

Keep in mind that on the GMAT, like in life, it is not always easy to identify the correct answer. So, if you can definitively eliminate four options, then the fifth option has to be the answer!

This leaves us with Option E. Most people would have found themselves stuck between D and E. This is why you need to analyse the options in detail so that you can understand why Option E is correct.

Let’s check!

At first glance, “deterrent effect” could make us think that this option is already telling us that increasing in the number of officers is not a good idea! But wait a minute!!! What does it mean to “have a deterrent effect on crime” mean? A deterrent is something that stops you from doing something. In other words, “have a deterrent effect on crime” means stop crime from happening.

Attachment:
wow-waaa.gif
wow-waaa.gif [ 1.82 MiB | Viewed 9790 times ]
In other words, this option is telling us that an increase in the number of officers will stop crime from happening! So, this option pretty much tells us that hiring ten new police officers is a good idea because it will stop crime from happening. Select!

Nitha Jay
GMAT Verbal SME
User avatar
Elite097
Joined: 20 Apr 2022
Last visit: 08 Oct 2025
Posts: 771
Own Kudos:
Given Kudos: 346
Location: India
GPA: 3.64
Posts: 771
Kudos: 553
Kudos
Add Kudos
Bookmarks
Bookmark this Post
ThatDudeKnows avigutman why not choice C? if they reduce support for other cities, they will have more money saved as a consequence which they can deploy to use here. So why is this not a weakener? I know the passage and the correct choice but I do not why C is not working out. Assumption also kind of hinges on the fact that funding is not enough for new people. But by reducing support for other services, clearly they can use those resources here .
User avatar
ThatDudeKnows
Joined: 11 May 2022
Last visit: 27 Jun 2024
Posts: 1,070
Own Kudos:
977
 [1]
Given Kudos: 79
Expert
Expert reply
Posts: 1,070
Kudos: 977
 [1]
1
Kudos
Add Kudos
Bookmarks
Bookmark this Post
Elite097
ThatDudeKnows avigutman why not choice C? if they reduce support for other cities, they will have more money saved as a consequence which they can deploy to use here. So why is this not a weakener? I know the passage and the correct choice but I do not why C is not working out. Assumption also kind of hinges on the fact that funding is not enough for new people. But by reducing support for other services, clearly they can use those resources here .

I don't read C that way. C suggests that the reason you'll need to reduce the other services is because the new police would be eating into the total available funds. You can't allocate those funds to police AND to something else.
User avatar
Elite097
Joined: 20 Apr 2022
Last visit: 08 Oct 2025
Posts: 771
Own Kudos:
553
 [1]
Given Kudos: 346
Location: India
GPA: 3.64
Posts: 771
Kudos: 553
 [1]
1
Kudos
Add Kudos
Bookmarks
Bookmark this Post
ThatDudeKnows
No. So the sequence is that they had a certain amount of funds available (say 1000). They say that this amount is sufficient to hire police but not the additional employees for conviction/ prison. Then they say it is foolish to hire more because we only have 1000 for these people and noone else. Fair. Now what we don't know is that they are also reducing funds elsewhere. Choice C tells us that we are reducing services elsewhere. So, once we know they are reducing service elsewhwere worth 500, we also get this additional amount so it does cast doubt on the fact that it is foolish to hire more people. Also there is nothing illogical about this thought process. Pls help clarify. Also this does not mean that they will be reducing other services for the new police because they already had sufficient funds for them to begin with. Where am I wrong and why


ThatDudeKnows
Elite097
ThatDudeKnows avigutman why not choice C? if they reduce support for other cities, they will have more money saved as a consequence which they can deploy to use here. So why is this not a weakener? I know the passage and the correct choice but I do not why C is not working out. Assumption also kind of hinges on the fact that funding is not enough for new people. But by reducing support for other services, clearly they can use those resources here .

I don't read C that way. C suggests that the reason you'll need to reduce the other services is because the new police would be eating into the total available funds. You can't allocate those funds to police AND to something else.
User avatar
ThatDudeKnows
Joined: 11 May 2022
Last visit: 27 Jun 2024
Posts: 1,070
Own Kudos:
Given Kudos: 79
Expert
Expert reply
Posts: 1,070
Kudos: 977
Kudos
Add Kudos
Bookmarks
Bookmark this Post
Elite097
ThatDudeKnows
No. So the sequence is that they had a certain amount of funds available (say 1000). They say that this amount is sufficient to hire police but not the additional employees for conviction/ prison. Then they say it is foolish to hire more because we only have 1000 for these people and noone else. Fair. Now what we don't know is that they are also reducing funds elsewhere. Choice C tells us that we are reducing services elsewhere. So, once we know they are reducing service elsewhwere worth 500, we also get this additional amount so it does cast doubt on the fact that it is foolish to hire more people. Also there is nothing illogical about this thought process. Pls help clarify and this does not mean that they will be reducing other services for the new police because they already had sufficient funds for them to begin with.


ThatDudeKnows
Elite097
ThatDudeKnows avigutman why not choice C? if they reduce support for other cities, they will have more money saved as a consequence which they can deploy to use here. So why is this not a weakener? I know the passage and the correct choice but I do not why C is not working out. Assumption also kind of hinges on the fact that funding is not enough for new people. But by reducing support for other services, clearly they can use those resources here .

I don't read C that way. C suggests that the reason you'll need to reduce the other services is because the new police would be eating into the total available funds. You can't allocate those funds to police AND to something else.

Where did you get 500 from. Using your numbers, it seems far more likely (and certainly possible) for that figure to just be the 100 that goes to the additional police. How did you decide that it should be a number greater than the amount that goes to the police.

At the end of the day, I think there’s only so much benefit that can be attained from picking apart an argument, question, or answer choice. You want to make sure that you learn whatever you can in order to learn the type of thinking, but arguing a specific question once you’ve already gotten what you can out of it doesn’t help further improve your score!

Posted from my mobile device
User avatar
avigutman
Joined: 17 Jul 2019
Last visit: 30 Sep 2025
Posts: 1,293
Own Kudos:
1,930
 [1]
Given Kudos: 66
Location: Canada
GMAT 1: 780 Q51 V45
GMAT 2: 780 Q50 V47
GMAT 3: 770 Q50 V45
Expert
Expert reply
GMAT 3: 770 Q50 V45
Posts: 1,293
Kudos: 1,930
 [1]
1
Kudos
Add Kudos
Bookmarks
Bookmark this Post
Elite097
Why not choice C? if they reduce support for other cities, they will have more money saved as a consequence which they can deploy to use here. So why is this not a weakener? I know the passage and the correct choice but I do not why C is not working out. Assumption also kind of hinges on the fact that funding is not enough for new people. But by reducing support for other services, clearly they can use those resources here .

(C) claims that approving the proposal would mean that support for other city services will have to be reduced during the next fiscal year. To me that seems to strengthen the claim that the proposal is foolish, not weaken it. I'm assuming, of course, that the other city services aren't currently overfunded. If they are, then I suppose (C) neither strengthens nor weakens the conclusion.

I think the error in your thinking, Elite097, is that you're evaluating whether the consequence of approving the proposal (support for other city services will have to be reduced during the next fiscal year) could cause the proposal to be not foolish, but that's circular logic. Let's abstract this reasoning:
X is a foolish proposal because it would lead to problem Y.
Answer choice (C) says: If X is approved, negative impact Z will occur.
If Z occurs, we can avoid problem Y (are you sure?)... Therefore, X isn't foolish after all.
User avatar
Elite097
Joined: 20 Apr 2022
Last visit: 08 Oct 2025
Posts: 771
Own Kudos:
553
 [1]
Given Kudos: 346
Location: India
GPA: 3.64
Posts: 771
Kudos: 553
 [1]
1
Kudos
Add Kudos
Bookmarks
Bookmark this Post
I think you did not follow my train of thought so let me clarify. It could be 50 or 100 to. That does not matter. Point is they already had 1000 and that was sufficient to accomodte police but not other employees. We do not know now whether they have another source of revenue too (in this case- reducing costs from other services). Now, in , we are told exactly this that getting funds for police would lead to reducing other services (which means we would get this additional amount of saving too whether it is 50 or 100 or 500 doesnt matter; all we care about is that we have this additional amount from savings too). So altogether we now have 1000 (Originally)+ 100 (from savings) and we already know 1000 is sufficient for the police so we can use this additional 100 for additional hires and hence it is not foolish after all to hire more. ThatDudeKnows
User avatar
Elite097
Joined: 20 Apr 2022
Last visit: 08 Oct 2025
Posts: 771
Own Kudos:
553
 [1]
Given Kudos: 346
Location: India
GPA: 3.64
Posts: 771
Kudos: 553
 [1]
1
Kudos
Add Kudos
Bookmarks
Bookmark this Post
Yes i think we can be sure to the extent of cast doubt on the conclusion. Conclusion says they are foolish cuz they have only sufficient funds but if we show them that we have even slightly more, it is enough to CAST doubt which is all we have to do in weaken and not break it completely right? avigutman. I am not sure what you used as a reason to think of it as a strengthener. Probably I am missing some details in what you wrote so if it is something with the verb or sequencing it might help if yu state it explicitly.

avigutman
Elite097
Why not choice C? if they reduce support for other cities, they will have more money saved as a consequence which they can deploy to use here. So why is this not a weakener? I know the passage and the correct choice but I do not why C is not working out. Assumption also kind of hinges on the fact that funding is not enough for new people. But by reducing support for other services, clearly they can use those resources here .

(C) claims that approving the proposal would mean that support for other city services will have to be reduced during the next fiscal year. To me that seems to strengthen the claim that the proposal is foolish, not weaken it. I'm assuming, of course, that the other city services aren't currently overfunded. If they are, then I suppose (C) neither strengthens nor weakens the conclusion.

I think the error in your thinking, Elite097, is that you're evaluating whether the consequence of approving the proposal (support for other city services will have to be reduced during the next fiscal year) could cause the proposal to be not foolish, but that's circular logic. Let's abstract this reasoning:
X is a foolish proposal because it would lead to problem Y.
Answer choice (C) says: If X is approved, negative impact Z will occur.
If Z occurs, we can avoid problem Y (are you sure?)... Therefore, X isn't foolish after all.
User avatar
ThatDudeKnows
Joined: 11 May 2022
Last visit: 27 Jun 2024
Posts: 1,070
Own Kudos:
Given Kudos: 79
Expert
Expert reply
Posts: 1,070
Kudos: 977
Kudos
Add Kudos
Bookmarks
Bookmark this Post
Elite097
I think you did not follow my train of thought so let me clarify. It could be 50 or 100 to. That does not matter. Point is they already had 1000 and that was sufficient to accomodte police but not other employees. We do not know now whether they have another source of revenue too (in this case- reducing costs from other services). Now, in , we are told exactly this that getting funds for police would lead to reducing other services (which means we would get this additional amount of saving too whether it is 50 or 100 or 500 doesnt matter; all we care about is that we have this additional amount from savings too). So altogether we now have 1000 (Originally)+ 100 (from savings) and we already know 1000 is sufficient for the police so we can use this additional 100 for additional hires and hence it is not foolish after all to hire more. ThatDudeKnows

Answer choice C: Hiring more police is going to reduce other services.
Argument: Hiring more police is foolish.

Given those two sentences, the road to making C weaken requires at least one additional "if" assumption. You and I disagree, but let's say we don't. Let's say C weakens. The question asks us which answer choice will most seriously weaken.

Per E, we have the money to hire the police, and hiring the police will be a deterrent on crime such that we may have no (or at least less than usual) additional court or prison expense. Wouldn't that make hiring the police a good thing, not "foolish?" Tell me why you eliminated E. If you do a good job with that, we can come back to C. ;)

The biggest take-away here is probably that if you find yourself needing to tell a long story to get to your answer choice, especially if you have an "if" in your story and ESPECIALLY if you have more than one "if," odds are that you've got the wrong answer choice.
User avatar
avigutman
Joined: 17 Jul 2019
Last visit: 30 Sep 2025
Posts: 1,293
Own Kudos:
1,930
 [2]
Given Kudos: 66
Location: Canada
GMAT 1: 780 Q51 V45
GMAT 2: 780 Q50 V47
GMAT 3: 770 Q50 V45
Expert
Expert reply
GMAT 3: 770 Q50 V45
Posts: 1,293
Kudos: 1,930
 [2]
2
Kudos
Add Kudos
Bookmarks
Bookmark this Post
Elite097
I am not sure what you used as a reason to think of it as a strengthener. Probably I am missing some details in what you wrote so if it is something with the verb or sequencing it might help if yu state it explicitly.
No problem, Elite097, I will try again.
Quote:
If funding for the new police officers’ salaries is approved, support for other city services will have to be reduced during the next fiscal year
This is an 'if' statement. What it tells us is that the proposal is even more foolish than we originally thought: it's not just that we won't be able to afford the salaries of additional court and prison employees... On top of that, we will have to reduce support for other city services.
I believe your confusion comes from a misinterpretation of what answer choice (C) is saying.
I think you're interpreting it in the following way:
(C) if funding for the new police officers' salaries is approved, the city will be able to reduce its spending on other city services during the next fiscal year.
Do you see the subtle difference? If you indeed had this interpretation in mind, you probably figured that thanks to the extra cops, we can save money in other areas, and use the savings as we see fit.
 1   2   
Moderators:
GMAT Club Verbal Expert
7445 posts
GMAT Club Verbal Expert
234 posts
188 posts